Identify the parent function for g(x) = (x + 3)^2 and describe the transformation.

Answers

Answer 1

The resulting graph of g(x) will resemble the graph of f(x), but it will be stretched vertically and pushed leftward by three units.

What is function?

Each element of a set (referred to as the domain) is mapped by a rule known as a function to a particular element of this other set (called the range). A function is, in other words, a connection between two subsets in which every member of the domain has a unique relationship to every element of the range. One popular approach to write a function is via function notation, which entails writing the function name surrounded by the incoming signal in parentheses, as in the following example: f (x). For instance, the formula f(x) = 2x + 1 takes the input x and produces the result 2x + 1.

given,

The fundamental quadratic function f(x) = x² serves as the parent function for g(x) = (x + 3)².

Since the argument of the function (x + 3) is x shifted left by 3 units, the graph of f(x) is horizontally displaced to the left by 3 units.

Given that the coefficient of the squared term is 1, the graph of the resulting function is shifted up by a factor of 1.

As a result, the transformation can be represented as a 3 unit horizontal shift and a 1 unit vertical stretch.

The resulting graph of g(x) will resemble the graph of f(x), but it will be stretched vertically and pushed leftward by three units.

To know more about function visit:

brainly.com/question/28193995

#SPJ1


Related Questions

Line segment RS is shown below with coordinates R(-8, -3)
and S(3, -3). Which coordinate below would represent R' if
point R was reflected across the x-axis?
A. (-8, 3)
B. (3, 3)
C. (8, -3)
D. (-3,-3)
10 9 8 7 6 5 4 3 2 1 1
R
2
4
-5
-6
10
•S.
S

Answers

Answer:

If point R is reflected across the x-axis, its y-coordinate will change sign. Therefore, the y-coordinate of R' will be 3 (the opposite of -3). Thus, the answer is A. (-8, 3)

Step-by-step explanation:

using graphical method to solve simultaneous equation y=2-2x and y=2x-6

Answers

The solution to the system of equations is x=2 and y=-2.

To solve the system of simultaneous equations graphically, we need to graph both equations on the same coordinate plane and find their point of intersection.

First, we'll rearrange both equations to be in the form y=mx+b, where m is the slope and b is the y-intercept.

y = 2 - 2x can be rewritten as y = -2x + 2

y = 2x - 6 can be rewritten as y = 2x - 6

Now, we'll plot both equations on the same coordinate plane. To do this, we'll create a table of values for each equation and plot the points.

For y = -2x + 2: (0,2), (1,0), (2,-2)

For y = 2x - 6:(0,-6), (1,-4), (2,-2)

Next, we'll plot these points on the same graph and draw the lines connecting them.

The point where the lines intersect is the solution to the system of equations. From the graph, we can see that the point of intersection is (2,-2).

To learn more about equations here:

https://brainly.com/question/29657983

#SPJ1

By about how much will g(x,y,z) = 3x + x COS Z-y sin z+y change if the point P(x,y,z) moves from P0(1.-3,0) a distance of ds= 0.1 unit toward the point P1(-1,-1,2)?

Answers

So the estimated value of √6.02 using differentials is approximately 2.4556.

The change in g(x,y,z) can be estimated using partial derivatives and differentials.

We can start by finding the partial derivatives of g(x,y,z) with respect to x, y, and z:∂g/∂x = 3 + cos(z)∂g/∂y = -sin(z) + 1∂g/∂z = -x sin(z) - y cos(z)Next, we can use the point P0(1,-3,0) and the distance ds = 0.1 to find the differentials dx, dy, and dz:dx = -2/√6 dsdy = 2/√6 dsdz = 1/√6 dsUsing these values, we can estimate the change in g:Δg ≈ (∂g/∂x) dx + (∂g/∂y) dy + (∂g/∂z) dzΔg ≈ (3 + cos(0)) (-2/√6 ds) + (-sin(0) + 1) (2/√6 ds) + (-1 sin(0) - (-3) cos(0)) (1/√6 ds)Δg ≈ (3 - 2/√6) dsPlugging in ds = 0.1, we get:Δg ≈ (3 - 2/√6) (0.1)Δg ≈ 0.389

Therefore, the change in g(x,y,z) is estimated to be approximately 0.389 units if the point P(x,y,z) moves from P0(1,-3,0) a distance of ds = 0.1 unit toward the point P1(-1,-1,2).

Suppose we want to estimate the value of √6.02 using differentials. We can start by choosing x = 6 and Δx = 0.02. Then, we need to find the derivative of f(x) = √x with respect to x:

f(x) = √x

f'(x) = 1/(2√x)

Using these values, we can estimate Δy:

Δy ≈ dy = f'(x) Δx

dy ≈ (1/(2√6)) (0.02)

dy ≈ 0.005

This means that a small change of 0.02 in x produces a small change of approximately 0.005 in y. To estimate the value of √6.02, we can add this change to the known value of √6:

√6.02 ≈ √(6 + 0.02) ≈ √6.04 ≈ 2.4556

For more similar questions on estimate

brainly.com/question/24431553

#SPJ11

Can you guess how many quarters TJ and Demi have in their pockets? And one Demi has nine more quarters than tj. And two if you double the number of quarters TJ has and triple the number of quarters then he has you would get 112 quarters in total. How many quarters does TJ have?​

Answers

Answer: TJ has 17 quarters and Demi has 26 quarters.

Step-by-step explanation:

Let t be the number of quarters TJ has and d be the number of quarters Demi has.

First, we will write an equation based on the first detail given:

       d = t + 9

Next, we will write an equation based on the second detail given:

       2t + 3d = 112

Now, we will substitute the first equation into the second and solve for t.

       2t + 3d = 112

       2t + 3(t + 9) = 112

       2t + 3t + 27 = 112

       5t + 27 = 112

       5t = 85

       t = 17 quarters

Lastly, we know that Demi has 9 more quarters than TJ. We will add 9.

       17 quarters + 9 quarters = 26 quarters

Answer:

TJ has 17 quarters.

Step-by-step explanation:

Let d and t equal the numbers of quarters Demi and TJ have, respectively.

"Demi has nine more quarters than TJ."

d = t + 9

"if you double the number of quarters TJ has and triple the number of quarters then he has you would get 112 quarters in total"

I think that "then he" above really should read "Demi."

2t + 3d = 112

d = t + 9

2t + 3d = 112

2t + 3(t + 9) = 112

2t + 3t + 27 = 112

5t = 85

t = 17

Answer: TJ has 17 quarters.

Five machines are cutting 1.25-foot long
metal sheets. The machines are being
calibrated to ensure that they are cutting
the accurate length. The previous batches
for each machine are shown in the table.
Select all of the statements that are valid
for the data.

Answers

Only the statement "One machine is considerably more unreliable than the rest." is valid for the data.

How to get the valid statements

Total number of correct cuts = 42 + 55 + 13 + 24 + 17 = 151

Total number of cuts = 100 + 100 + 100 + 100 + 100 = 500

Percentage of correct cuts = (151/500) * 100 = 30.2%

This statement is not valid, as only 30.2% of the cuts are the correct length.

One machine is considerably more unreliable than the rest."

By examining the number of correct cuts for each machine, we can see that Machine 3 has only 13 correct cuts, while the other machines have more than 17. This statement is valid.

3. When a machine misses the correct length, it tends to cut too long."

We need to compare the number of cuts that are too long (1.26-1.27 feet) with those that are too short (1.23-1.24 feet) across all machines:

Total number of cuts too long = 4 + 2 + 3 + 6 + 4 = 19

Total number of cuts too short = 980 + 72 + 67 = 1119

This statement is not valid, as the machines tend to cut too short rather than too long.

4. "Machine 5 will cut every batch the correct length at least 92% of the time."

To check this statement, we need to find the percentage of correct cuts for Machine 5:

Percentage of correct cuts for Machine 5 = (17/100) * 100 = 17%

This statement is not valid, as Machine 5 only cuts the correct length 17% of the time, which is less than 92%.

only the statement "One machine is considerably more unreliable than the rest." is valid for the data.

Read more on valid statements here:https://brainly.com/question/28335524

#SPJ1

Pls help quick
which theorem can you use to show that the quadrilateral on the tile floor is a parallelogram

Answers

To show that the quadrilateral on the tile floor is a parallelogram, you can use the opposite sides theorem, opposite angles theorem, consecutive angles theorem, and Diagonal bisector theorem.


1. Opposite sides theorem: If both pairs of opposite sides of the quadrilateral are congruent (equal in length), then it is a parallelogram.

2. Opposite angles theorem: If both pairs of opposite angles of the quadrilateral are congruent (equal in measure), then it is a parallelogram.

3. Consecutive angles theorem: If the consecutive angles of the quadrilateral are supplementary (their sum is 180 degrees), then it is a parallelogram.

4. Diagonal bisector theorem: If the diagonals of the quadrilateral bisect each other (divide each other into two equal parts), then it is a parallelogram.

Choose the most appropriate theorem based on the given information and apply it to prove that the quadrilateral is a parallelogram.

Learn more about parallelograms:

https://brainly.com/question/970600

#SPJ11

Problem 7. (1 point) Suppose you are given a solid whose base is the circle x2 + y2 = 36 and the cross sections perpendicular to the x- axis are triangles whose height and base are equal. Find the area of the vertical cross section A at the level X = 3.

Answers

The shape formed by a solid intersecting with a plane, so the At level X = 3, the area of the vertical cross-section A is 108 square units.

To find the area of the vertical cross section A at the level X = 3, we need to find the equation of the circle when it is intersected by the plane X = 3.
First, let's find the value of y when X = 3 using the equation of the circle x^2 + y^2 = 36:

(3)^2 + y^2 = 36
9 + y^2 = 36
y^2 = 27
y = ±√27

Since we are dealing with a circle, there are two points on the circle at X = 3, which are (3, √27) and (3, -√27).

The distance between these two points will be the base of the triangle, which is also equal to its height (as given in the problem).

Base and height of the triangle: 2 * √27

Now we can find the area A of the vertical cross-section, which is a triangle with equal base and height:

A = 1/2 * base * height
A = 1/2 * (2 * √27) * (2 * √27)
A = 4 * 27
A = 108

So, the area of the vertical cross-section A at the level X = 3 is 108 square units.

Visit here to learn more about circle:

brainly.com/question/3033257

#SPJ11

A plumber charges 14.95 to come to the house and 27.50 per hour the plumber sends a 138.70 bill

Answers

The plumber worked for 4.5 hours and charged $138.70 for their services

How we find the time plumber work?

To find out how many hours the plumber worked, we first need to subtract the initial charge of $14.95 from the total bill of $138.70.

$138.70 - $14.95 = $123.75

This gives us the amount that the plumber charged for the hours worked. Now, we can divide this amount by the hourly rate to find the number of hours:

$123.75 ÷ $27.50 per hour = 4.5 hours

However, we need to convert the decimal part (0.5) into minutes. We can do this by multiplying it by 60:

0.5 x 60 = 30 minutes

But we need to add the initial charge of $14.95 to get the final answer.

4 hours and 30 minutes is equivalent to 4.5 hours.

4.5 hours x $27.50 per hour = $123.75

$123.75 + $14.95 = $138.70

Learn more about Hourly rate

brainly.com/question/29335545

#SPJ11

On your own paper, make a frequency table for and find the mean to the nearest hundredth. 6. 7, 6, 6, 7, 6, 5, 8, 6, 5, 9, 8, 5, 6, 8 9, 5, 8, 8, 6, 8, 7, 5, 6,9,7,7,9,6 7. 501 501

Answers

After drawing our frequency table, we also find out that our mean is 6.73.

How to make a frequency table and find the mean?

To make a frequency table, we have to count the number of times each value appears in the data set.

Frequency table:

Value       Frequency

5              4

6              8

7              4

8              6

9              3

To find the mean, we will add all values and divide by total number of values. The mean is:

= EF / N

= (6 + 7 + 6 + 6 + 7 + 6 + 5 + 8 + 6 + 5 + 9 + 8 + 5 + 6 + 8 + 9 + 5 + 8 + 8 + 6 + 8 + 7 + 5 + 6 + 9 + 7 + 7 + 9 + 6 + 7) / 30

= 6.83333333333

= 6.83.

Read more about frequency table

brainly.com/question/27820465

#SPJ4

what inequality does the graph represent

Answers

Answer:

  (c)  y < -x +1

Step-by-step explanation:

You want the inequality that matches the graph.

Boundary line

The boundary line of the shaded area has a negative slope: it falls one unit for each unit to the right, so the slope is ...

  m = rise/run = -1/1 = -1

The line crosses the y-axis at y = 1, so the y-intercept is b = 1.

The equation of the boundary line is ...

  y = mx + b

  y = -x +1

Shading

The boundary line is dashed, so its values are not part of the solution set. The shading is below the line, so only y-values less than those on the line are included.

The inequality is ...

  y < -x +1 . . . . . choice C

<95141404393>

A random sample of 155 observations results in 62 successes. [You may find it useful to reference the z table.]a. Construct the a 90% confidence interval for the population proportion of successes. (Round intermediate calculations to at least 4 decimal places. Round "z" value and final answers to 3 decimal places.)b. Construct the a 90% confidence interval for the population proportion of failures. (Round intermediate calculations to at least 4 decimal places. Round "z" value and final answers to 3 decimal places.)

Answers

For a random sample of 155 observations results in 62 successes.

a) A 90% confidence interval for the population proportion of successes is equals to the (0.335 , 0.465).

b) A 90% confidence interval for the population proportion of failure is equals to the (0.535 , 0.665).

We have a random sample of 155 observations results in 62 successes. So,

Observed value, x = 62

Sample size,n = 155

Population Proportion, p = x/n

= 62/155

= 0.4

a) We have to determine 90% confidence interval for the population proportion of successes. Using the distribution table, for 90% confidence interval, z-score value is equals to 1.6. Consider Confidence interval formula with proportion, CI [tex]= p ± z×\sqrt\frac{p(1-p)}{n}[/tex]

substitute all known values in above formula, [tex]= 0.4 ± 1.64\sqrt\frac{0.4(1- 0.4)}{155}[/tex]

= 0.4 ± 0.0645

= (0.4 - 0.0645 , 0.4 + 0.0645)

= (0.335 , 0.465)

b) Now, we have to determine a 90% confidence interval for the population proportion of failures.

Now consider, here failure observed values, x = 155 - 62

= 93

proportion, p = x/n

= 93/155 = 0.6

Consider the confidence interval formula, CI [tex]= p ± z×\sqrt\frac{p(1-p)}{n}[/tex]

substitute values, [tex]= 0.6 ±1.64×\sqrt\frac{0.6(1-0.6)}{155}[/tex]

= 0.6 ± 0.0645

= (0.6 - 0.0645 , 0.6 + 0.0645)

= (0.535 , 0.665)

Hence, required value is (0.535 , 0.665).

For more information about confidence interval, visit :

https://brainly.com/question/17212516

#SPJ4

A paper bag has five colored marbles. The marbles are red, green, blue, yellow, and orange. List the sample space when choosing one marble.
S = {1, 2, 3, 4, 5}
S = {red, blue, green, yellow}
S = {g, r, b, y, o}
S = {green, blue, yellow, orange}

Answers

The sample space for choosing one marble from the given marbles which are red, green, blue, yellow, and orange is s = {red, green, blue, yellow, orange}

Given color of the marbles which are present in the paper bag are,

red  greenblueyelloworange

The five color marbles are present in the paper bag so, the sample space also contains all five marbles without repeating any color again.

Sample space: sample space is nothing but listing all the outcomes of the event. In the above event, we have to list the all outcomes when choosing one marble from the paper bag which containing five different marbles.

So, the sample space S = {red, green, blue, yellow, orange}

To know more about sample spaces,

https://brainly.com/question/29719992

#SPJ1

The American Heart Association is about to conduct an anti-smoking campaign and wants to know the fraction of Americans over 40 who smoke. Step 2 of 2: Suppose a sample of 1089 Americans over 40 is drawn. Of these people, 806 don't smoke. Using the data, construct the 85% confidence interval for the population proportion of Americans over 40 who smoke. Round your answers to three decimal places

Answers

To construct a confidence interval for the population proportion of Americans over 40 who smoke, we can use the formula:

Confidence Interval = Sample Proportion ± (Critical Value) x Standard Error

where the sample proportion is the number of individuals who don't smoke divided by the total sample size (806/1089), the critical value can be found using a normal distribution table or calculator with the given confidence level (85%), and the standard error can be calculated using the formula:

Standard Error = √[ (Sample Proportion x (1 - Sample Proportion)) / Sample Size ]

Plugging in the given values, we get:

Sample Proportion = 806/1089 = 0.740
Sample Size = 1089
Standard Error = √[(0.740 x 0.260) / 1089] = 0.016
Critical Value (using a normal distribution table or calculator) = 1.440

Therefore, the 85% confidence interval for the population proportion of Americans over 40 who smoke is:

0.740 ± (1.440 x 0.016) = 0.740 ± 0.023

Rounding to three decimal places, the confidence interval is (0.717, 0.763). This means that we can be 85% confident that the true proportion of Americans over 40 who smoke falls between 71.7% and 76.3%.

To know more about Standard Error:

https://brainly.com/question/1191244

#SPJ11

Find the magnitude of v. v = 7i
|lv|| = _____

Answers

The magnitude of vector v is:
|v| = 7

How magnitude of vector is calculated?

The magnitude of v is simply the length of the vector v, which can be found using the Pythagorean theorem. The vector v is given as v = 7i.

To find the magnitude of v (|v|), use the formula:
|v| = √(x² + y²)

where x and y are the components of the vector v. In this case, x = 7 (from 7i) and y = 0 (since there is no j component).

Now, plug in the values of x and y into the formula:
|v| = √(7² + 0²)
|v| = √(49 + 0)
|v| = √(49)
|v| = 7

Learn more about Pythagorean theorem.

brainly.com/question/14930619

#SPJ11

Help with problem with photo

Answers

Check the picture below.

Find the domain and range of the function V(x, y) = 9√9y – 45x^2. Indicate the domain of V in equality or inequality notation. Use <= to denote ≤ and >= to denote ≥.
Domain of V = {(2,y) }

Answers

The minimum value of 9y – 45x^2 is 0, which occurs when y = 5x^2/3, so the range of V is all non-negative real numbers:

Range of V: [0, ∞)

To find the domain and range of the function V(x, y) = 9√(9y – 45x^2), we need to consider the values of x and y that make the expression under the square root non-negative, since we cannot take the square root of a negative number.

So, we have:

9y – 45x^2 >= 0

Dividing both sides by 9 and rearranging, we get:

y >= 5x^2/3

This means that the domain of V is all points (x, y) such that y is greater than or equal to 5x^2/3:

Domain of V: {(x, y) | y >= 5x^2/3}

To find the range of V, we note that the square root is always non-negative, so V(x, y) will be non-negative whenever 9y – 45x^2 is non-negative. The minimum value of 9y – 45x^2 is 0, which occurs when y = 5x^2/3, so the range of V is all non-negative real numbers:

Range of V: [0, ∞)

To learn more about domain visit:https://brainly.com/question/28135761

#SPJ11

PLS ANSWER QUICK
The table shows the length, in inches, of fish in a pond.


11 19 9 15
7 13 15 28


Determine if the data contains any outliers. If so, list the outliers.
There is an outlier at 28.
There is an outlier at 7.
There are outliers at 7 and 28.
There are no outliers.

Answers

Answer:

There is an outlier at 28.

Triangle ABC is similar to triangle DBE. Select the responses that make the statements true. Large triangle A B C with side length 7. 5. Smaller triangle D B C inside A B C, which shares vertex B. Side B E has length 5 and base D E has length 13

Answers

The correct responses are: "Triangle DBC is similar to triangle ABC" and "The length of side DE is 13."

Since triangle ABC is similar to triangle DBE, we know that the corresponding angles are congruent and the corresponding sides are proportional.

From the given information, we know that side BC of triangle ABC corresponds to side BE of triangle DBE, since they share vertex B. Therefore, we can use the proportion:

BC / BE = AC / DE

Substituting the given values, we have:

BC / 5 = 7.5 / 13

Solving for BC, we get:

BC = (5 x 7.5) / 13 = 2.88 (rounded to two decimal places)

Therefore, the length of side BC is 2.88.

Now we can check which of the given statements are true:

"The length of side AB is 3.75." We do not have enough information to determine the length of side AB, so this statement cannot be determined to be true or false based on the given information.

"Triangle DBC is similar to triangle ABC." This statement is true, since they share angle B and the sides BC and BE are proportional.

"Angle C in triangle ABC is congruent to angle D in triangle DBE." This statement cannot be determined to be true or false based on the given information, since we do not know which angle in triangle DBE corresponds to angle C in triangle ABC.

"The length of side AC is 4.29." This statement cannot be determined to be true or false based on the given information, since we only have information about side BC and side BE. We do not have enough information to determine the length of side AC.

"The length of side DE is 7.8." This statement is false, since the length of side DE is given as 13, not 7.8.

To learn more about Triangle here:

https://brainly.com/question/14926756

#SPJ4

PLEASE HELPPPPPPPPPPPPPPPPPPPPPPPPPPPPPPPPPPPPPPPPPPPPPPPPPPPPPPPPPPPPPPPPPPPPPPPPPPPPPPPPPP

Answers

Answer:true

Step-by-step explanation:

Answer:

true

Step-by-step explanation

what is 88 closer to 64 or 125

Answers

Answer:

64

Step-by-step explanation:

Answer:

64...i think

Step-by-step explanation:

125 - 88=37

88-64=24

PLS MARK BRAINLIEST

Point B is the image of point A when point A is rotated about the origin. What is known about point A and B?

Answers

Point B is the image of point A under a rotation about the origin.

Describe Rotation?

Rotation is a transformation in which an object or a point is turned around a fixed point or a fixed axis. The fixed point or axis is called the center of rotation, and the angle of rotation specifies the amount and direction of the turn. When an object is rotated, its orientation changes but its shape and size remain the same.

For example, if you rotate a square by 90 degrees around its center, it will look the same as before, but it will be oriented differently. Similarly, if you rotate a point in a coordinate system around the origin, its position will change, but its distance from the origin will remain the same.

Rotations are commonly used in geometry, physics, and engineering to describe the motion of objects, such as the rotation of the Earth around its axis or the rotation of a wheel on an axle. They are also used in computer graphics to create animations and in robotics to control the movement of robotic arms and other devices.

When point A is rotated about the origin to form point B, the distance between the origin and each point remains the same. The direction from the origin to point A and the direction from the origin to point B are related by the angle of rotation. Specifically, point B is the image of point A under a rotation about the origin.

To know more about transformation visit:

https://brainly.com/question/12221775

#SPJ1

what is the simplified form of the expression below (3m^4n)^3(2m^2n^5p)/6m^4n^9p^8

Answers

For the expression (3m⁴n)³(2m²n⁵p)/6m⁴n⁹p⁸, the simplified-value is 9m¹⁰n⁻¹p⁻⁷.

To simplify the expression (3m⁴n)³(2m²n⁵p)/6m⁴n⁹p⁸, we first use the exponent-rule that states (qᵃ)ᵇ = qᵃᵇ to simplify the first part of the expression:

⇒ (3m⁴n)³ = 3³(m⁴)³n³ = 27m¹²n³;

Next, we can simplify the denominator by using the rules of exponents to combine the like terms:

⇒ 6m⁴n⁹p⁸ = 2×3m⁴n⁹p⁸;

Substituting the values,

We get;

⇒ (27m¹²n³)×(2m²n⁵p)/(2*3m⁴n⁹p⁸);

Simplifying the expression by cancelling out the common factors, we get:

⇒ 9m¹⁰n⁻¹p⁻⁷;

Therefore, the simplified-value is : 9m¹⁰n⁻¹p⁻⁷.

Learn more about Expression here

https://brainly.com/question/29139540

#SPJ1

The given question is incomplete, the complete question is

What is the simplified form of the expression below (3m⁴n)³(2m²n⁵p)/6m⁴n⁹p⁸;

Hello! Help please thank you ​

Answers

Answer:

2(2(3) + 2(5) + 3(5)) = 2(6 + 10 + 15) = 2(31)

= 62

D is the correct answer.

The outside temperature was 4°C for the next six hours the temperature changed at a mean rate of -0. 8°C per hour for the next two hours what was the final temperature

Answers

The final temperature after the next 8 hours (6 hours at -0.8°C per hour, followed by 2 hours at -0.8°C per hour) will be -2.4°C.

The final temperature can be calculated by subtracting the total temperature change from the initial temperature of 4°C.

The total temperature change during the next six hours can be calculated by multiplying the mean rate of -0.8°C per hour by the number of hours, which is 6.

-0.8°C/hour x 6 hours = -4.8°C

Therefore, the temperature after the next six hours will be:

4°C - 4.8°C = -0.8°C

For the next two hours, the temperature changed at a mean rate of -0.8°C per hour. This means the temperature decreased by:

-0.8°C/hour x 2 hours = -1.6°C

So the final temperature will be:

-0.8°C - 1.6°C = -2.4°C.

Therefore, the final temperature after the next 8 hours (6 hours at -0.8°C per hour, followed by 2 hours at -0.8°C per hour) will be -2.4°C.

To learn more about temperature, refer below:

https://brainly.com/question/11464844

#SPJ11

How many cubes wit a side length of 1/2 foot could he fit Inside the box .

How many cubes wit a side length of 1/8 foot could he fit inside the box

Answers

Answer:

12

325

Step-by-step explanation:

When it mentions the word "fit", you use TSA.

TSA cuboid = 2[LH+BH+LH] = 2 [ (2×3/2)+(3/2×7/2)+(2×7/2)] = 2 [ 3 + 21/4 + 7] = 2 [5¼+10] = 2 [15¼] = 2 × 61/4 = 61/2 = 30½ ft²

TSA ½ foot cube = 6L² = 6(½)² = 6×¼ = 2½

Number of cubes = 30.5÷2.5 = 12.2 = 12

TSA ⅛ foot cube = 6(⅛)² = 6×1/64 = 3/32

Number of cubes = 30.5÷3/32 = 325⅓ = 325

Please help I need this done ASAP

Answers

Answer:

Domain is all x values

Range is all y values

Step-by-step explanation:

Your image is not clear enough for me to see the x or y coordinates so hope that helps you to figure it out on your own

(2^-1/2) / (2^1/2)

How to flip negative exponents

Answers

The value of the expression is 2

What are index forms?

Index forms are described as those forms that are used to represent numbers that are too large or small in more convenient forms.

They are also described as numbers that are raised to a variable or an exponents.

Other names for index forms are scientific notations and standard forms.

One of the rules of index forms is that the exponents are added when the have the same and are being multiplied.

From the information given, we have that;

(2^-1/2) / (2^1/2)

subtract the exponents

2^-1/2-1/2

subtract the values

2^ -1

Then, we have;

2

Learn about index forms at: https://brainly.com/question/15361818

#SPJ1

What is the average rate of change of the function g(x)=6x from x=-1 to x=3? show your work or explain how you obtained your response

Answers

The average rate of change of the function g(x)=6x from x=-1 to x=3 is found to be 6.

The function g(x) = 6x describes a relationship between x and the value of 6 times x. We want to find the average rate of change of this function from x = -1 to x = 3. The average rate of change tells us the average amount by which the function changes per unit of change in x over this interval.

In this case, by using the function g(x) = 6x and evaluating it for x = 3 and x = -1, a difference of 18 - (-6) = 24 is found. The difference in x's values is equal to 3 - (-1) = 4. We divide these to get an average rate of change of 6.

To know more about average rate change, visit,

https://brainly.com/question/2170564

#SPJ4

if f' * (x) = 2x - 1 and g(x) - x + 3 prove that f g(x) is a linear function​

Answers

The composite function fg(x) is a linear function​ by the proof shown below

Proving that the function fg(x) is a linear function​

From the question, we have the following parameters that can be used in our computation:

f(x) = 2x - 1

g(x) = -x + 3

The above functions are linear functions

This means that the function fg(x) will also be a linear function​

To prove this, we have

f(g(x)) = 2(g(x)) - 1

substitute the known values in the above equation, so, we have the following representation

f(g(x)) = 2(-x + 3) - 1

So, we have

f(g(x)) = -2x - 7

Hence, the function is a linear function

Read more about composite functions at

https://brainly.com/question/10687170

#SPJ1

From 9 names on a ballot, a committee of 5 will be elected to attend a political national convention. How many different committees are possible? Use the empirical probability formula to solve the exercise

Answers

The number of different committees that are possible is: 126

How to solve Permutation and Combination?

When you have a number of elements and then want to form subsets of elements of particular smaller size, you can utilize combinations or permutations. If the order of placement of the elements does not matter, we use combinations to quantify the number of groups formed.

Now, the order of the members in the committee does not matter and as such we will use combinations which has the formula:

nCr = n!/(r!(n - r)!)

We are given:

n = 9

r = 5

Thus:

9C5 = 9!/(5!(9 - 5)!)

= 126 different committees

Read more about Permutation and Combination at: https://brainly.com/question/4658834

#SPJ4

Other Questions
How are the sentiments of the Seafarer similar to those of the Wanderer? How are they different? In a tube-within-a-tube body plan, what is the interior tube? What statement best completes the diagram enlightenment ideas are introduced in Europe and the second continental congress ratified the declaration of independence? Lily's age is 2 years and 4 months.Hugo's age is 1 year and 8 months.Write Lily's age in months as a fraction of Hugo's age in months.Give your answer in it's simplest form. A 2-quart carton of pineapple juice costs $8.08. What is the price per cup?$ Decision tree:1. Suppose Mr. Abdullah has $50,000 to invest in the financial market for one year. His choices havebeen narrowed to two options. Assume that any long-term capital gains will be taxed at 20%. Mr. Abdullahs minimum attractive rate of return (MARR) is known to be 5% after taxes. Determine thepayoff amount at the tip of each branch. Option 1. Buy 1,000 shares of a technology stock at $50 per share that will be held for oneyear. Since this is a new initial public offering (IPO), there is not much research informationavailable on the stock; hence, there will be a brokerage fee of $100 for this size ofthe transaction (for either buying or selling stocks). Assume that the stock is expected to providea return at any one of three different levels: a high level (A) with a 50% return ($25,000), amedium level (B) with a 9% return ($4,500), or a low level (C) with a 30% loss Assume alsothat the probabilities of these occurrences are assessed at 0. 25, 0. 40, and 0. 35, respectively. No stock dividend is anticipated for such a growth-oriented company. Option 2. Purchase a $50,000 U. S. Treasury bond, which pays interest at an effective annualrate of 7. 5% ($3,750). The interest earned from the Treasury bond is nontaxable income. However, there is a $150 transaction fee for either buying or selling the bond. Mr. Abdullahs dilemma is which alternative to choose to maximize his financial gain PLEAASE PLEASE HELP ME!!! I'm failing science and he gave us this test to do. PLEASE HELP. I'll give lots of pointsYou are tasked with investigating the sudden die-off of salmon in a river in Washington State. The salmon were trying to swim upstream to spawn but died near or just past a dam about halfway along the river's path. You observe that numerous area farms apply atrazine to their crops at this time of year and that the one dam does employ methods for allowing salmon to move upstream past it. The water quality data you collected include:- high levels of sediment at one location near the middle of the river's path by the dam- low levels of the toxin atrazine at some of the upstream testing locations along the river- very low levels of dissolved oxygen along the entire downstream half of the river due to recent algae buildupWhich type of water pollution seems MOST LIKELY to have contributed to the sudden death of the salmon and why? Include a description of your chosen type of water pollution A research group is designing a home to be heated with passive solar heating. This allows the home to be heated using solar energy that radiates into the home through windows. What scientific principle could the research group utilize in order to maximize the efficiency of their design? A)Thinner window panes will retain more heat in the summer than thicker window panes. EliminateB)Latex paints will reflect light more efficiently than enamel paints used on the walls. C)Darker flooring materials will absorb and retain solar energy more efficiently than lighter materials. D)Energy efficient light bulbs will utilizer less electricity than incandescent bulbs used in the lamps and other lighting fixtures The age discrimination in employment act specifically outlaws hiring practices that discriminate against people:_____.a. who are between 39-69. b. who are under the age of 18.c. who are 40 and over. d. who are between 16 and 20. e. who are younger than 18 and older than 39. What is the distance in feet between the contour lines in the diagram? A. 5 B. 10 C. 15 11. The spread of a drop of food dye throughout a glass of milk is called__________ Write the linear equation that gives the rule for this table.X4567Y24303642Write your answer as an equation with y first, followed by an equals sign. Victoria has $200 of her birthday gift money saved at home, and the amount is modeled by the function h(x) = 200. She reads about a bank that has savings accounts that accrue interest according to the function s(x) = (1. 05)x1. Explain how Victoria can combine the two functions to model the total amount of money she will have in her bank account as interest accrues after she deposits her $200. Justify your reasoning. WILL GIVE BRANLIEST Write an essay that synthesizes information from at least three (3) sentences to develop your position on the use of video games in the educational environment. (100 POINTs will give BRAINIEST FOR the amount of EFFORT you put into it) Please I need helpSympathy, Paul Lawrence Dunbar 1. What mood does Dunbar create through his use of imagery to describe the setting in this stanza? How do you know? 2. How does the caged bird contrast with this use of imagery? How might the caged bird be developed as a symbol?3. What are some possible themes of this poem? Whose perspective is beingpresented in the followingpassage?36- As for Nicholas, he, too,was silent, in the absorption ofone who has much to thinkabout; it was just possible, heconsidered, that the huntsmanwould escape with his houndswhile the wolves feasted onthe stricken stag.A. The Aunt'sB. The cousins'C. Nicholas' Ying Yu bought a rectangular box to display her doll collection. She decidedto exchange the box for a similar one that had five times its dimensions. How does the volume of the larger rectangular box compare to the volumeof the smaller box? Is these cosine or tangent or sine ? I need help with them can somebody tell me the answer to all three What technique is I pressed my face into the snow gritted my teeth and waited How did the Latin American population change between the years 1500 and 1600? Why?